Stay ahead of learning milestones! Enroll in a class over the summer!

G
Topic
First Poster
Last Poster
Italian WinterCamps test07 Problem4
mattilgale   88
N 12 minutes ago by OronSH
Source: ISL 2006, G3, VAIMO 2007/5
Let $ ABCDE$ be a convex pentagon such that
\[ \angle BAC = \angle CAD = \angle DAE\qquad \text{and}\qquad \angle ABC = \angle ACD = \angle ADE.
\]The diagonals $BD$ and $CE$ meet at $P$. Prove that the line $AP$ bisects the side $CD$.

Proposed by Zuming Feng, USA
88 replies
mattilgale
Jan 29, 2007
OronSH
12 minutes ago
Hard to approach it !
BogG   129
N 16 minutes ago by OronSH
Source: Swiss Imo Selection 2006
Let $\triangle ABC$ be an acute-angled triangle with $AB \not= AC$. Let $H$ be the orthocenter of triangle $ABC$, and let $M$ be the midpoint of the side $BC$. Let $D$ be a point on the side $AB$ and $E$ a point on the side $AC$ such that $AE=AD$ and the points $D$, $H$, $E$ are on the same line. Prove that the line $HM$ is perpendicular to the common chord of the circumscribed circles of triangle $\triangle ABC$ and triangle $\triangle ADE$.
129 replies
BogG
May 25, 2006
OronSH
16 minutes ago
Simple triangle geometry [a fixed point]
darij grinberg   48
N 20 minutes ago by OronSH
Source: German TST 2004, IMO ShortList 2003, geometry problem 2
Three distinct points $A$, $B$, and $C$ are fixed on a line in this order. Let $\Gamma$ be a circle passing through $A$ and $C$ whose center does not lie on the line $AC$. Denote by $P$ the intersection of the tangents to $\Gamma$ at $A$ and $C$. Suppose $\Gamma$ meets the segment $PB$ at $Q$. Prove that the intersection of the bisector of $\angle AQC$ and the line $AC$ does not depend on the choice of $\Gamma$.
48 replies
darij grinberg
May 18, 2004
OronSH
20 minutes ago
IMO ShortList 1998, geometry problem 4
orl   14
N 22 minutes ago by OronSH
Source: IMO ShortList 1998, geometry problem 4
Let $ M$ and $ N$ be two points inside triangle $ ABC$ such that
\[ \angle MAB = \angle NAC\quad \mbox{and}\quad \angle MBA = \angle NBC.
\]
Prove that
\[ \frac {AM \cdot AN}{AB \cdot AC} + \frac {BM \cdot BN}{BA \cdot BC} + \frac {CM \cdot CN}{CA \cdot CB} = 1.
\]
14 replies
orl
Oct 22, 2004
OronSH
22 minutes ago
Functional equation
Nima Ahmadi Pour   98
N 37 minutes ago by ezpotd
Source: ISl 2005, A2, Iran prepration exam
We denote by $\mathbb{R}^+$ the set of all positive real numbers.

Find all functions $f: \mathbb R^ + \rightarrow\mathbb R^ +$ which have the property:
\[f(x)f(y)=2f(x+yf(x))\]
for all positive real numbers $x$ and $y$.

Proposed by Nikolai Nikolov, Bulgaria
98 replies
Nima Ahmadi Pour
Apr 24, 2006
ezpotd
37 minutes ago
Geometry
noneofyou34   0
an hour ago
Please can someone help me prove that orthocenter of a triangle exists by using Menelau's Theorem!
0 replies
noneofyou34
an hour ago
0 replies
Hard combi
EeEApO   0
an hour ago
In a quiz competition, there are a total of $100 $questions, each with $4$ answer choices. A participant who answers all questions correctly will receive a gift. To ensure that at least one member of my family answers all questions correctly, how many family members need to take the quiz?

Now, suppose my spouse and I move into a new home. Every year, we have twins. Starting at the age of $16$, each of our twin children also begins to have twins every year. If this pattern continues, how many years will it take for my family to grow large enough to have the required number of members to guarantee winning the quiz gift?
0 replies
EeEApO
an hour ago
0 replies
Inequality with mathematical means
StefanSebez   12
N an hour ago by Sh309had
Source: Serbia JBMO TST 2022 P1
Prove that for all positive real numbers $a$, $b$ the following inequality holds:
\begin{align*}
\sqrt{\frac{a^2+b^2}{2}}+\frac{2ab}{a+b}\ge \frac{a+b}{2}+ \sqrt{ab}
\end{align*}When does equality hold?
12 replies
StefanSebez
Jun 1, 2022
Sh309had
an hour ago
Really fun geometry problem
Sadigly   4
N an hour ago by Double07
Source: Azerbaijan Senior MO 2025 P6
In the acute triangle $ABC$ with $AB<AC$, the foot of altitudes from $A,B,C$ to the sides $BC,CA,AB$ are $D,E,F$, respectively. $H$ is the orthocenter. $M$ is the midpoint of segment $BC$. Lines $MH$ and $EF$ intersect at $K$. Let the tangents drawn to circumcircle $(ABC)$ from $B$ and $C$ intersect at $T$. Prove that $T;D;K$ are colinear
4 replies
1 viewing
Sadigly
2 hours ago
Double07
an hour ago
Orthocenter
jayme   8
N an hour ago by cj13609517288
Dear Mathlinkers,

1. ABC an acuatangle triangle
2. H the orthcenter of ABC
3. DEF the orthic triangle of ABC
4. A* the midpoint of AH
5. X the point of intersection of AH and EF.

Prove : X is the orthocenter of A*BC.

Sincerely
Jean-Louis
8 replies
jayme
Mar 25, 2015
cj13609517288
an hour ago
Constructing graphs satisfying conditions on degrees
jlammy   19
N an hour ago by de-Kirschbaum
Source: EGMO 2017 P4
Let $n\geq1$ be an integer and let $t_1<t_2<\dots<t_n$ be positive integers. In a group of $t_n+1$ people, some games of chess are played. Two people can play each other at most once. Prove that it is possible for the following two conditions to hold at the same time:

(i) The number of games played by each person is one of $t_1,t_2,\dots,t_n$.

(ii) For every $i$ with $1\leq i\leq n$, there is someone who has played exactly $t_i$ games of chess.
19 replies
jlammy
Apr 9, 2017
de-Kirschbaum
an hour ago
An easy geometry in Taiwan TST
Li4   6
N an hour ago by wassupevery1
Source: 2022 Taiwan TST Round 3 Independent Study 1-G
Let $ABC$ be an acute triangle with orthocenter $H$ and circumcircle $\Omega$. Let $M$ be the midpoint of side $BC$. Point $D$ is chosen from the minor arc $BC$ on $\Gamma$ such that $\angle BAD = \angle MAC$. Let $E$ be a point on $\Gamma$ such that $DE$ is perpendicular to $AM$, and $F$ be a point on line $BC$ such that $DF$ is perpendicular to $BC$. Lines $HF$ and $AM$ intersect at point $N$, and point $R$ is the reflection point of $H$ with respect to $N$.

Prove that $\angle AER + \angle DFR = 180^\circ$.

Proposed by Li4.
6 replies
Li4
Apr 27, 2022
wassupevery1
an hour ago
Factorising and prime numbers...
Sadigly   3
N an hour ago by cj13609517288
Source: Azerbaijan Senior MO 2025 P4
Prove that for any $p>2$ prime number, there exists only one positive number $n$ that makes the equation $n^2-np$ a perfect square of a positive integer
3 replies
Sadigly
3 hours ago
cj13609517288
an hour ago
Do not try to overthink these equations
Sadigly   3
N 2 hours ago by cj13609517288
Source: Azerbaijan Senior MO 2025 P2
Find all the positive reals $x,y,z$ satisfying the following equations: $$y=\frac6{(2x-1)^2}$$$$z=\frac6{(2y-1)^2}$$$$x=\frac6{(2z-1)^2}$$
3 replies
1 viewing
Sadigly
3 hours ago
cj13609517288
2 hours ago
Vertices of a convex polygon if and only if m(S) = f(n)
orl   12
N Apr 24, 2025 by Maximilian113
Source: IMO Shortlist 2000, C3
Let $ n \geq 4$ be a fixed positive integer. Given a set $ S = \{P_1, P_2, \ldots, P_n\}$ of $ n$ points in the plane such that no three are collinear and no four concyclic, let $ a_t,$ $ 1 \leq t \leq n,$ be the number of circles $ P_iP_jP_k$ that contain $ P_t$ in their interior, and let \[m(S)=a_1+a_2+\cdots + a_n.\]Prove that there exists a positive integer $ f(n),$ depending only on $ n,$ such that the points of $ S$ are the vertices of a convex polygon if and only if $ m(S) = f(n).$
12 replies
orl
Aug 10, 2008
Maximilian113
Apr 24, 2025
Vertices of a convex polygon if and only if m(S) = f(n)
G H J
Source: IMO Shortlist 2000, C3
The post below has been deleted. Click to close.
This post has been deleted. Click here to see post.
orl
3647 posts
#1 • 6 Y
Y by Adventure10, junioragd, and 4 other users
Let $ n \geq 4$ be a fixed positive integer. Given a set $ S = \{P_1, P_2, \ldots, P_n\}$ of $ n$ points in the plane such that no three are collinear and no four concyclic, let $ a_t,$ $ 1 \leq t \leq n,$ be the number of circles $ P_iP_jP_k$ that contain $ P_t$ in their interior, and let \[m(S)=a_1+a_2+\cdots + a_n.\]Prove that there exists a positive integer $ f(n),$ depending only on $ n,$ such that the points of $ S$ are the vertices of a convex polygon if and only if $ m(S) = f(n).$
This post has been edited 1 time. Last edited by djmathman, Oct 3, 2016, 3:25 AM
Reason: changed formatting to match imo compendium
Z K Y
The post below has been deleted. Click to close.
This post has been deleted. Click here to see post.
Thjch Ph4 Trjnh
205 posts
#2 • 4 Y
Y by Adventure10, Mango247, and 2 other users
$ f(n) = 2.(_4^n)$
Z K Y
The post below has been deleted. Click to close.
This post has been deleted. Click here to see post.
Myth
4464 posts
#3 • 4 Y
Y by Ali3085, Adventure10, Mango247, and 1 other user
It is strange to see such an easy and evident problem in IMO SL.
Z K Y
The post below has been deleted. Click to close.
This post has been deleted. Click here to see post.
SnowEverywhere
801 posts
#4 • 12 Y
Y by Catalanfury, Vaijan_Mama, k12byda5h, DCMaths, Adventure10, Mango247, Stuffybear, winniep008hfi, and 4 other users
We claim that the function $f(n)=2 \binom{n}{4}$ satisfies the requirements.

Let the score $s(a,b,c,d)$ of the four points $P_a$, $P_b$, $P_c$ and $P_d$ be the number of points $P_i$ where $i \in \{a,b,c,d \}$ such that $P_i$ is properly contained in the circle passing through the remaining three points. Observe that

\[m(S) = \sum_{1 \le a<b<c<d \le n} s(a,b,c,d)\]

First we prove the following lemma.

Lemma 1. The score of a convex quadrilateral $2$.

Proof. Let the vertices of the convex quadrilateral be denoted as $A, B, C$ and $D$. We have that point $D$ lies within the circumcircle of $\triangle{ABC}$ if and only if

\[\angle{ABC} > 180 - \angle{ADC} \quad \Leftrightarrow \quad \angle{ABC} + \angle{ADC} > 180\]

Therefore if $D$ lies within the circumcircle of $\triangle{ABC}$, it also follows by symmetry that $B$ lies within the circumcircle of $\triangle{ADC}$. If not, then since the sum of the interior angles of $ABCD$ is $360$,

\[\angle{ABC} + \angle{ADC} < 180 \quad \Rightarrow \quad \angle{BAD} + \angle{ACD} > 180\]

Therefore $A$ lies within the circumcircle of $\triangle{BCD}$ and $C$ lies within the circumcircle of $\triangle{ABD}$. In both cases, the score of $ABCD$ is equal to $2$.

Lemma 2. The score of a concave quadrilateral is $1$.

Proof. Let $ABCD$ denote the concave quadrilateral. Without the loss of generality, let $A$ be such that interior angle $\angle{BAC} > 180$. It follows that $A$ lies in the interior of triangle $\triangle{BCD}$. Therefore, the circumcircle of $\triangle{BCD}$ contains $A$. However, none of the remaining three circles passing through three of the points $A, B, C$ and $D$ contain the remaining point. Hence the score of $ABCD$ is equal to $1$.

If Direction. If the vertices of $S$ form a convex $n$-gon, then each quadrilateral formed by four distinct points in $S$ is a convex quadrilateral and therefore

\[m(S) = \sum_{1 \le a<b<c<d \le n} s(a,b,c,d) = 2 \binom{n}{4}\]

Only If Direction. If the vertices of $S$ form a concave $n$-gon, then at least one of the quadrilaterals formed by four distinct points in $S$ is a concave quadrilateral and therefore

\[m(S) = \sum_{1 \le a<b<c<d \le n} s(a,b,c,d) < 2 \binom{n}{4}\]

The function $f(n)=2 \binom{n}{4}$ therefore satisfies that $m(S)=f(n)$ if and only if the points in $S$ are the vertices of a convex polygon.
Z K Y
The post below has been deleted. Click to close.
This post has been deleted. Click here to see post.
JackXD
151 posts
#5 • 1 Y
Y by Adventure10
Lemma Every quadrilateral $S$ satisfies $m(S) \le 2$ with equality if and only if it is convex,

Proof:If a quadrilateral $ABCD$ is convex then one out of $\angle{ABC}+\angle{ADC}$ and $\angle{DAB}+\angle{DCB}$ is greater than $\pi$ and the other is less than $\pi$.This implies $m(S)=2$.One the other hand if it is concave then clearly $m(S)=1$


Back to our main problem.If S forms a convex polygon then every quadrilateral $P_{i}P_{j}P_{k}P_{l}$ contributes $1$ to two of $a_{i},a_{j},a_{k}$ and $a_{l}$ (from the lemma) and thus contributes two to $m(S)$.This immediately implies $f(n)=2\binom{n}{4}$

Now let $m(S)=f(n)=2\binom{n}{4}$.Each quadrilateral $P_{i}P_{j}P_{k}P_{l}$ contributes atmost $2$ to $f(n)$ and thus $m(S) \le 2\binom{n}{4}$.As there is equality,we have that each quadrilateral contributes exactly $2$ to $f(n)$,hence from the lemma every quadrilateral is convex,implying that $S$ forms a convex polygon.
This post has been edited 3 times. Last edited by JackXD, Jan 7, 2016, 4:22 PM
Reason: xx
Z K Y
The post below has been deleted. Click to close.
This post has been deleted. Click here to see post.
william122
1576 posts
#6 • 2 Y
Y by Adventure10, Mango247
Note that given any 4 points, they add 2 to the count if their convex hull is a quadrilateral, and 1 otherwise. So, $f(n)=2\binom{n}{2}$, with equality achieved iff all quadruplets of points form convex quadrilaterals. However, if there exists a point $P_i$ inside the convex hull $Q_1,Q_2,\ldots,Q_k$, then it must be in one of the triangles $Q_1Q_2Q_3$, $Q_1Q_3Q_4,\ldots Q_1Q_{k-1}Q_k$, which cover the convex hull. So, there exist 4 points whose convex hull is a triangle. Therefore, the only way equality is reached is if all points are on the convex hull, as desired.
Z K Y
The post below has been deleted. Click to close.
This post has been deleted. Click here to see post.
niyu
830 posts
#7
Y by
The key idea is to consider four-tuples.

Lemma: If $ABCD$ is a non-cyclic convex quadrilateral, $m(ABCD) = 2$.

Proof: Suppose $ABCD$ is convex and non-cyclic. Note that $\angle ABC + \angle ADC \neq \angle BCD + \angle BAD \neq 180^\circ$. Hence, exactly one of these two sums is greater than $180^\circ$. WLOG, suppose $\angle ABC + \angle ADC > 180^\circ$ and $\angle BCD + \angle BAD < 180^\circ$. Since $\angle ABC > 180^\circ - \angle ADC$ it follows that $B$ lies within $(ADC)$. Similarly, $D$ lies within $(ABC)$. Meanwhile, since $\angle BCD < 180^\circ - \angle BAD$, it follows that $A$ does not lie within $(BCD)$, and similarly, $C$ does not lie within $(ABD)$. This implies that $m(ABCD) = 2$, proving the lemma. $\blacksquare$

Lemma: If $ABCD$ is a concave quadrilateral, $m(ABCD) = 1$.

Proof: Say $\angle BAD > 180^\circ$. Since $A$ and $C$ both lie on the same side of $\overline{BD}$, and $\angle BCD < \angle BAD$, it follows that $C$ does not lie within $(ABD)$, while $A$ lies within $(BCD)$. Meanwhile, since $\angle ABC + \angle ADC < 180^\circ$, by the same argument as in the previous lemma we find that $B$ does not lie within $(ACD)$ and that $D$ does not lie within $(ABC)$. Hence, $m(ABCD) = 1$. $\blacksquare$

We now return to the given problem. We claim that $m(S) \leq 2\binom{n}{4}$, and that equality holds iff the points of $S$ form a convex quadrilateral. Indeed, as no four points in $S$ are concyclic, we have
\begin{align*}
	m(S) &= \sum_{1 \leq w < x < y < z \leq n} m(P_wP_xP_yP_z) \\
	&\leq \sum_{1 \leq w < x < y < z \leq n} 2 \\
	&\leq 2\binom{n}{4}.
\end{align*}Equality holds here iff each of the quadrilaterals $P_wP_xP_yP_z$ is convex, which occurs iff the points of $S$ form a cyclic quadrilateral. This completes the proof. $\Box$
Z K Y
The post below has been deleted. Click to close.
This post has been deleted. Click here to see post.
bluelinfish
1449 posts
#8
Y by
Rather easy for a C3.

All angles are in degrees.

Claim: Suppose $Q$ consists of four points $A,B,C,D$. Then $m(Q)$ is $2$ if $A,B,C,D$ form a convex quadrilateral and $1$ otherwise.
Proof. Notice that if $A,B,C,D$ form a convex quadrilateral, then $D$ appears inside the circumcircle of $ABC$, $\angle D$ must be greater than $180-\angle B$, which is equivalent to $\angle B + \angle D >180$. Since exactly one opposite pair of angles sum to greater than $180$ degrees, there will be exactly two points that are contained in the circumcircle of the other three.

If $A,B,C,D$ are not convex, WLOG let $A,B,C$ be the convex hull. Then it is clear that the only point that is contained in the circumcircle of the other three is $D$. $\blacksquare$

The key step is to notice that $$m(S)=\sum_{1\le a<b<c<d\le n} m\left(\{P_a,P_b,P_c,P_d\}\right)$$because both quantities count the amount of ordered pairs containing a single point of $S$ and a set of three points in $S$, with all four points distinct, such that the single point is inside the circumcircle of the three points.

Using our claim, $m(S)$ must be equal to twice the number of four-point subsets of $S$ that consist of points forming a convex quadrilateral plus the four-point subsets of $S$ that do not. Moreover, every subset of four points form a convex quadrilateral iff $S$ does, so $m(S)=2\binom{n}{4}$ iff $S$ forms a convex quadrilateral. We are done.
This post has been edited 1 time. Last edited by bluelinfish, Jan 18, 2022, 10:45 PM
Z K Y
The post below has been deleted. Click to close.
This post has been deleted. Click here to see post.
awesomeming327.
1714 posts
#9 • 3 Y
Y by Mango247, Mango247, Mango247
Let $m(\{a,b,c,d\})$ be the number $m(S)$ for the quadrilateral $P_aP_bP_cP_d.$ It is easy to see that $m(S)$ is the sum of $m(\{a,b,c,d\})$ for all choices of subset $\{a,b,c,d\} \subseteq S$. Note that $(P_aP_bP_c)$ contains $P_d$ if only $P_d$ lies inside of the angle $P_aP_bP_c$ and $\angle P_b+\angle P_d> 180^\circ.$ Clearly, when $P_aP_bP_cP_d$ then $m(a,b,c,d)=2$ and when it is nonconvex $m(a,b,c,d)=1$. Therefore, $m(S)=2\tbinom{n}4$ if and only if $S$ is convex.
Z K Y
The post below has been deleted. Click to close.
This post has been deleted. Click here to see post.
john0512
4187 posts
#10
Y by
Note that by swapping the order of summation, $m(S)$ is equal to the number of quadruples of points $(A,B,C,D)$ where $D$ is inside $(ABC)$ and $A,B,C$ are unordered.

Claim: Each set of 4 points contributes $2$ if they are convex, and $1$ if they are not.

If $ABCD$ is convex and non-cyclic, then we either have $\angle A+\angle C>180$ or $\angle B+\angle D>180$ but not both. However, since $A$ is inside $(BCD)$ if and only if $\angle A+\angle C>180$, etc, the convex quadriateral contributes $2$. If $D$ is inside $\triangle ABC$, then $D$ will be inside $(ABC)$ but nothing else works.

Thus, the maximum possible value of $m(S)$ is $2{n\choose 4}$, with equality if and only if each quadrilateral is convex, which is the same as saying the entire set is convex. We are done as $f(n)=2{n\choose 4}$.
Z K Y
The post below has been deleted. Click to close.
This post has been deleted. Click here to see post.
asdf334
7585 posts
#11
Y by
For any four points $S'=\{P_a,P_b,P_c,P_d\}$ count the value $m(S')$. Notice this is either $2$ if the convex hull is a quadrilateral (i.e. the points form a convex polygon) and $1$ if the convex hull is a triangle (i.e. there is an interior point).
Clearly $f(n)$ is the sum of $m(S')$ over all such $S'$. Hence the maximum occurs if every convex hull of four points is a quadrilateral. If the points of $S$ are the vertices of a convex polygon this occurs. If the points of $S$ are not the vertices of a convex polygon then triangulate the convex hull. Any interior point is contained in a triangle and for these four points we have $m(S')=1$. $\blacksquare$
Z K Y
The post below has been deleted. Click to close.
This post has been deleted. Click here to see post.
onyqz
195 posts
#12
Y by
storage
solution
Z K Y
The post below has been deleted. Click to close.
This post has been deleted. Click here to see post.
Maximilian113
575 posts
#13
Y by
Observe that in any $4$ points $A, B, C, D,$ if they form a convex polygon there are $2$ pairs of the form $(i, \omega)$ where $i$ is a point from $A, B, C, D$ and $\omega$ is the circumcircle of the other points. However if they form a non-convex polygon there is $1$ only.

Clearly, if we consider all quadruples of points from our $n$ points, and sum up the number of valid pairs, every point, along with a circumcircle it lies in, is counted. In addition, the point and circumcircle pair uniquely determines which quadruple it was counted in, meaning that this count yields a injection and surjection to $m(S),$ so there is a bijection.

Therefore if $x$ quadruples are convex and $y$ are concave, $$m(S) = 2x+y.$$But $x+y=\binom{n}{4}$ so $$m(S)=\binom{n}{4}+x,$$and the desired result follows. QED
Z K Y
N Quick Reply
G
H
=
a